Akademisyenler öncülüğünde matematik/fizik/bilgisayar bilimleri soru cevap platformu
0 beğenilme 0 beğenilmeme
3.5k kez görüntülendi
1.$\lim\limits_{n\to\infty}r^n=0\Leftrightarrow|r|<1$

2.$|r|>{1}\Rightarrow\lim\limits_{n\to\infty}r^n\not\in\mathbb{R}$

Olduğunu nasıl gösterebiliriz? 1. kuralın doğruluğu sezgilerle hissedilebiliyor ama 2. için çok emin olamadım. Bunlar ispatlanabilir mi? Ayrıca ikinci kural için 1'i dahil edebilir miyiz? Site içerisinde birbirinden farklı yorumlar gördüm bununla alakalı.
Orta Öğretim Matematik kategorisinde (194 puan) tarafından  | 3.5k kez görüntülendi

Ilki icin hissel olarak $r=1/2$ ve ikincisi icinse $r=2$ orneklerini dusunebilirsin.

Ayrica $1^n=1$ her $n$ pozitif tam sayisi icin gecerlidir.

Hocam o zaman

$\lim\limits_{n\to\infty}\left(1+\dfrac{1}{n}\right)^n$

İfadesinde de 1'e eşit olması gerekmez mi?

İçin limiti 1e gitmiyor. İç direkt kendisi 1.

$r=-1$ için $|r|=1$ oluyor ama (1e yakınsayan bir alt dizisi ve $-1$ e yakınsayan başka bir alt dizisi var olduğu için) $\lim\limits_{n\to\infty}(-1)^n$ yoktur.

20,200 soru
21,728 cevap
73,275 yorum
1,887,975 kullanıcı